[Math] Evaluating $\sum_{n=0}^{\infty}{\arctan(n+2)-\arctan(n)}$

calculussequences-and-series

The above is the telescoping series and the terms cancel but I'm left with $-\arctan(1)$ which is equal to $-\pi \over 4$ which is not correct.

Best Answer

There are more terms surviving after telescoping. Namely, we are left with \begin{align}\lim_{n \rightarrow \infty} \Bigl(\arctan (n+1) + \arctan n - \arctan 1 - \arctan 0\Bigr) = \frac{ \pi}{2} +\frac{ \pi}{2} -\frac{ \pi}{4}-0 =\frac{ 3\pi}{4}. \end{align}